LSAT 92 – Section 1 – Question 12

You need a full course to see this video. Enroll now and get started in less than a minute.

Target time: 1:12

This is question data from the 7Sage LSAT Scorer. You can score your LSATs, track your results, and analyze your performance with pretty charts and vital statistics - all with a Free Account ← sign up in less than 10 seconds

Question
QuickView
Type Tags Answer
Choices
Curve Question
Difficulty
Psg/Game/S
Difficulty
Explanation
PT92 S1 Q12
+LR
Necessary assumption +NA
A
0%
146
B
2%
150
C
6%
149
D
91%
161
E
1%
147
132
139
147
+Easier 147.037 +SubsectionMedium

This is an NA question.

Sometimes, when the structure of the argument is very simple, NA answers turn out to also be SA answers. Here, the question stem says “properly drawn,” which hints that the answer might also be an SA answer.

The stimulus begins with OPA. Health experts who advocate moderate consumption of alcohol for its cardiac benefits say that red wine is the most beneficial. Next, we get "but" and “while.” Both indicate transition. “While” indicates a transition to a concession point. The author concedes that there is statistical evidence that people who drink moderate amounts of red wine are less likely to fall victim to heart disease than are people who drink moderate amounts of other alcoholic beverages. So that’s her concession to OPA on red wine’s benefit.

Now we get back to the “but.” What the author really wants to say is that the proponents of red wine have yet to determine what biochemical mechanism, if any, explains this difference. So, the author concludes, their evidence is inconclusive.

Okay, that’s a very simple argument. One premise, one conclusion. Why is the OP’s evidence inconclusive? Because OP have not determined a biochemical mechanism.

Well, we’ve seen this kind of NA question before. We need a P to C bridge. We need to connect the idea in the premise to the idea in the conclusion.

And that’s what Correct Answer Choice (D) does. (D) says that evidence regarding the health benefits of foods or drinks must be corroborated by accurate accounts of the mechanisms involved in order to be conclusive. Great, just what we needed. For evidence to be conclusive, there must be an accurate account of the mechanisms involved. The premise fails the necessary condition (OP gave no account at all of the mechanism). Therefore, contrapose and we can conclude that OP’s evidence is not conclusive.

This is a classic NA question where the correct answer builds a P to C bridge. If this were a PSA question or SA question, (D) would still be the correct answer. That’s not always the case, but sometimes the correct answer in an NA question overlaps with the correct answer in a PSA or SA question.

Answer Choice (A) says if a substance has health benefits when consumed in moderation, then physicians should advocate moderate consumption even if excess consumption is harmful. The argument is not about what physicians should or shouldn't advocate. We’re not talking about what physicians ought to do. We're talking about whether there is enough evidence that red wine consumption is beneficial.

Answer Choice (C) suffers from the same issue. Physicians should not make any recommendations regarding something… We don't care.

The author is not writing an op-ed telling people to drink more or less red wine. Rather, the author is making an argument about whether the people who are claiming that red wine is the most beneficial of alcohols for heart health benefits (OP) have a good argument. And she's saying that they don't.

If (C) isn’t necessary, does it connect at all to the argument? If we take our author's conclusion to be true, that is, evidence is inconclusive about whether red wine is healthy, and if we take (C) to be true, then we can infer that physicians shouldn't make any recommendations.

Answer Choice (B) says assertions regarding the health benefits of foods or drinks should be supported by persuasive statistical evidence. Not necessary. The argument isn’t concerned about statistical evidence. The author already conceded that there is some statistical evidence in support of OPA.

If (B) is negated, the author’s argument still stands as long as (D) is true.

Answer Choice (E) says one should try to find a causal mechanism for the health benefits of red wine only if the health benefits have been conclusively demonstrated. This a mishmash of the words and concepts that we might expect to find in the correct answer choice. The argument just assumes that if you don't have a causal mechanism, then you don't have conclusive evidence. That's it. (E) is trying to mimic those concepts and relationships. (E) is saying if health benefits haven’t been conclusively demonstrated, then you shouldn’t try to find a causal mechanism for those as-of-yet-undemonstrated health benefits. I get the feeling that (E) is worried about wasting time and that’s why (E) is admonishing against prematurely rushing to find a causal mechanism. (E) is saying first make sure that the health benefits have been conclusively demonstrated.

Take PrepTest

Review Results

Leave a Reply